Đến nội dung

Hình ảnh

Chuyên đề Phần nguyên


  • Please log in to reply
Chủ đề này có 42 trả lời

#21
hxthanh

hxthanh

    Tín đồ $\sum$

  • Hiệp sỹ
  • 3921 Bài viết
Dưới đây tôi xin trình bày lại định lý số 4 trong Chuyên Đề Phần Nguyên
$\begin{array}{|c|}
\hline
{\boxed{\textbf{Định lý 4}}}\\
\begin{array}{l}
\text{Cho } p \text{ là một số nguyên tố lẻ } q \text{ là số nguyên không chia hết cho } p.\\
\text{Giả sử hàm } f:\mathbb{N}^* \longrightarrow \mathbb{R} \text{ thỏa mãn đồng thời hai điều kiện sau:}\\
\bullet\quad\dfrac{f\left(k\right)}{p} \notin \mathbb{Z},\quad \forall k=1,2,...,p-1\\
\bullet\quad\dfrac{f\left(k\right)+f\left(p-k\right)}{p} \in \mathbb{Z},\quad \forall k=1,2,...,p-1\\
\text{Khi đó ta có: }\quad\sum\limits_{k=1}^{p-1}\left\lfloor f(k)\dfrac{q}
{p}\right\rfloor=\dfrac{q}{p}\sum\limits_{k=1}^{p-1}f(k) -\dfrac{p-1}{2}\\
\end{array}\\
\hline
\end{array}$

Định lý này phát biểu khá dài dòng và khó nhớ, nhưng ứng dụng của nó rất đáng được quan tâm.
Chính vì vậy tôi sẽ đưa ra điểm mấu chốt nhằm giúp các bạn dễ hiểu và áp dụng được định lý
này vào các bài toán tính tổng phần nguyên.

Mấu chốt của vấn đề đó là:
$\begin{array}{| |}
\hline
\text{Nếu } x,y \text{ là hai số không nguyên, nhưng } x+y \in \mathbb{Z} \text{ thì ta có: }\\
\lfloor x\rfloor + \lfloor y\rfloor = x+y-1 \\
\hline
\end{array}$

Chẳng hạn như: $\lfloor 1,4 \rfloor + \lfloor 1,6\rfloor =1,4+1,6-1=2$
Chứng minh tính chất này rất đơn giản
Đặt $x=\lfloor x \rfloor + \{x\};\quad y=\lfloor y \rfloor + \{y\}$ suy ra $x+y=\lfloor x \rfloor+\lfloor y \rfloor+\{x\}+\{y\}\in \mathbb{Z}$
Do $0<\{x\};\{y\}<1\quad\Rightarrow \{x\}+\{y\}=1$
Vậy ta có điều cần chứng minh

Trở lại với định lý 4

Với mọi $1\le k \le p-1$, ta có:
$f(k)\dfrac{q}{p}\notin\mathbb{Z}$ và $f(p-k)\dfrac{q}{p}\notin\mathbb{Z}$ Lại có: $f(k)\dfrac{q}{p} + f(p-k)\dfrac{q}{p} \in\mathbb{Z}$
Suy ra: $\left\lfloor f(k)\dfrac{q}{p}\right\rfloor + \left\lfloor f(p-k)\dfrac{q}{p}\right\rfloor = f(k)\dfrac{q}{p} + f(p-k)\dfrac{q}{p} - 1$
Lấy tổng từ $k=1$ đến $p-1$ đẳng thức trên ta được

$\quad\sum\limits_{k=1}^{p-1} \left(\left\lfloor f(k)\dfrac{q}{p}\right\rfloor + \left\lfloor f(p-k)\dfrac{q}{p}\right\rfloor\right) = \sum\limits_{k=1}^{p-1}\left(f(k)\dfrac{q}{p} + f(p-k)\dfrac{q}{p} - 1\right)$
$\Leftrightarrow 2\sum\limits_{k=1}^{p-1}\left\lfloor f(k)\dfrac{q}{p}\right\rfloor = 2 \sum\limits_{k=1}^{p-1} f(k)\dfrac{q}{p} - (p-1)$

Từ đó suy ra kết luận của định lý.

#22
hxthanh

hxthanh

    Tín đồ $\sum$

  • Hiệp sỹ
  • 3921 Bài viết

$\boxed{Bt2.19}$
Cho $p$ là số nguyên tố lẻ, $q$ là số nguyên không chia hết cho $p$.
Chứng minh rằng:
$\sum\limits_{k=1}^{p-1} \left\lfloor (-1)^k k^2\dfrac{q}{p}\right\rfloor = \dfrac{(p-1)(q-1)}{2}$

Lời giải:

Ta có:
$2\sum\limits_{k=1}^{p-1} \left\lfloor (-1)^k k^2\dfrac{q}{p}\right\rfloor = \sum\limits_{k=1}^{p-1}\left(\left\lfloor (-1)^k k^2\dfrac{q}{p}\right\rfloor+\left\lfloor (-1)^{p-k} (p-k)^2\dfrac{q}{p}\right\rfloor\right)\qquad (1)$

Nhận xét: Do $p$ là số nguyên tố lẻ và $q$ không chia hết cho $p$
Nên $(-1)^k k^2\dfrac{q}{p} \notin \mathbb{Z},\quad \forall 1\le k\le p-1$
Mặt khác: $(-1)^k k^2\dfrac{q}{p}+(-1)^{p-k} (p-k)^2\dfrac{q}{p}=(-1)^k\left(k^2-(p-k)^2\right)\dfrac{q}{p}=(-1)^{k+1}q(p-2k)\in \mathbb{Z}$

Do đó:
$\left\lfloor (-1)^k k^2\dfrac{q}{p}\right\rfloor+\left\lfloor (-1)^{p-k} (p-k)^2\dfrac{q}{p}\right\rfloor=(-1)^k k^2\dfrac{q}{p}+(-1)^{p-k} (p-k)^2\dfrac{q}{p}-1=(-1)^{k+1}q(p-2k)-1$

Thay vào $(1)$ ta được

$2\sum\limits_{k=1}^{p-1} \left\lfloor (-1)^k k^2\dfrac{q}{p}\right\rfloor =\sum\limits_{k=1}^{p-1}\left((-1)^{k+1}q(p-2k)-1\right)=-\,(p-1)+\sum\limits_{k=1}^{p-1}(-1)^{k+1}q(p-2k)=$
$=-\,(p-1)+\sum\limits_{r=1}^{\frac{p-1}{2}}q(p-4r+2)-\sum\limits_{r=1}^{\frac{p-1}{2}}q(p-4r)$

$($Chia tổng đan dấu thành 2 nửa, nửa đầu là các số hạng thứ tự lẻ $(k=2r-1)$, nửa còn lại là các số hạng có thứ tự chẵn $(k=2r))$

$\Rightarrow 2\sum\limits_{k=1}^{p-1} \left\lfloor (-1)^k k^2\dfrac{q}{p}\right\rfloor =-\,(p-1)+\sum\limits_{r=1}^{\frac{p-1}{2}}2q=(p-1)(q-1)$

$\Rightarrow \sum\limits_{k=1}^{p-1} \left\lfloor (-1)^k k^2\dfrac{q}{p}\right\rfloor =\dfrac{(p-1)(q-1)}{2}$

$\blacksquare$

#23
Mitsuru

Mitsuru

    Lính mới

  • Thành viên
  • 5 Bài viết
Thầy Thanh có thể giải giúp em bài 1.11 và bài toán sau được không ạ:

Chứng minh rằng với mọi số thực dương $x$ và số nguyên dương $n$, ta luôn có:

$\left [ nx \right ] \geq \left [ x \right ] + \frac{\left [ 2x \right ]}{2} + \frac{\left [ 3x \right ]}{3} + ... + \frac{\left [ nx \right ]}{n}$

Em cảm ơn thầy nhiều :)

Bài viết đã được chỉnh sửa nội dung bởi Mitsuru: 10-06-2012 - 16:24

Đi có thể không đến.... Nhưng không đi thì sẽ KHÔNG BAO GIỜ đến!


#24
hxthanh

hxthanh

    Tín đồ $\sum$

  • Hiệp sỹ
  • 3921 Bài viết

Thầy Thanh có thể giải giúp em bài 1.11 và bài toán sau được không ạ: Chứng minh rằng với mọi số thực dương $x$ và số nguyên dương $n$, ta luôn có: $\left [ nx \right ] \geq \left [ x \right ] + \frac{\left [ 2x \right ]}{2} + \frac{\left [ 3x \right ]}{3} + ... + \frac{\left [ nx \right ]}{n}$ Em cảm ơn thầy nhiều :)


Bài toán của em đưa ra là bài USAMO số 5 năm 1981
$\begin{array}{|c|}
\hline
\fbox{USAMO 1981 #5}\\
\text{Chứng minh rằng với } n\in \mathbb{N}^* \text{ và số thực dương } x \text{ ta có bất đẳng thức:}\\
S_n(x)=\sum\limits_{k=1}^n \dfrac{\left\lfloor kx\right\rfloor}{k} \le \left\lfloor nx \right\rfloor\quad(1)\\
\hline
\end{array}$
Có nhiều cách giải cho bài toán này, một trong số đó là lời giải (hay nhất) bằng phương pháp quy nạp.
Lời giải: (quy nạp)
Với $n=1$ đẳng thức xảy ra. Vậy $(1)$ đúng!
Giả sử $(1)$ đúng với $1,2,...,n-1$, ta chứng minh $(1)$ cũng đúng với $n$
Ta có:
$\begin{eqnarray}
S_1(x)&=&\left\lfloor x\right\rfloor \\
2S_2(x)-2S_1(x)&=&\left\lfloor 2x\right\rfloor\\
3S_3(x)-3S_2(x)&=&\left\lfloor 3x\right\rfloor\\
\vdots\qquad\qquad\vdots\quad&=&\quad\vdots\quad\\
nS_n(x)-nS_{n-1}(x)&=&\left\lfloor nx\right\rfloor\\
\end{eqnarray}$

Cộng $n$ đẳng thức trên lại ta có:
$\Rightarrow nS_n(x)-\sum\limits_{k=1}^{n-1} S_k(x)=\sum\limits_{k=1}^n \left\lfloor kx\right\rfloor$
$\Rightarrow nS_n(x)=\sum\limits_{k=1}^{n-1} S_k(x)+\sum\limits_{k=1}^n \left\lfloor kx\right\rfloor$
$\Rightarrow nS_n(x) \le \sum\limits_{k=1}^{n-1} \left\lfloor kx\right\rfloor + \sum\limits_{k=1}^n \left\lfloor kx\right\rfloor\quad\text{(giả thiết quy nạp)}$
$\Rightarrow nS_n(x) \le \sum\limits_{k=1}^{n-1} \left\lfloor (n-k)x\right\rfloor + \sum\limits_{k=1}^{n-1} \left\lfloor kx\right\rfloor+\left\lfloor nx\right\rfloor\quad\text{(đảo chiều tổng thứ nhất, tách số hạng cuối tổng thứ 2)}$
$\Rightarrow nS_n(x) \le \left\lfloor nx\right\rfloor +\sum\limits_{k=1}^{n-1}\left(\left\lfloor (n-k)x\right\rfloor +\left\lfloor kx\right\rfloor\right) $
$\Rightarrow nS_n(x) \le \left\lfloor nx\right\rfloor +\sum\limits_{k=1}^{n-1}\left\lfloor (n-k)x+kx\right\rfloor\quad\text{(Dùng BĐT: }\left\lfloor x\right\rfloor+\left\lfloor y\right\rfloor \le \left\lfloor x+y\right\rfloor\text{)}$
$\Rightarrow nS_n(x) \le \left\lfloor nx\right\rfloor+(n-1)\left\lfloor nx\right\rfloor$
$\Rightarrow S_n(x) \le \left\lfloor nx\right\rfloor$

Vậy $(1)$ cũng đúng với $n$, theo nguyên lý quy nạp ta có điều cần chứng minh!

#25
hxthanh

hxthanh

    Tín đồ $\sum$

  • Hiệp sỹ
  • 3921 Bài viết

$\boxed{Bt1.11}$
Chứng minh rằng: $\sum\limits_{k=0}^{+\infty} \left\lfloor \dfrac{x+2^k}{2^{k+1}}\right\rfloor = \left\lfloor x \right\rfloor$

Xét tổng sau:
$\quad S_n=\sum\limits_{k=0}^n \left\lfloor \dfrac{x+2^k}{2^{k+1}}\right\rfloor$
$\Rightarrow S_n=\sum\limits_{k=0}^n \left\lfloor \dfrac{x}{2^{k+1}}+\dfrac{1}{2}\right\rfloor$
$\Rightarrow S_n=\sum\limits_{k=0}^n \left(\left\lfloor \dfrac{x}{2^k}\right\rfloor-\left\lfloor \dfrac{x}{2^{k+1}}\right\rfloor\right)\quad\text{(Theo định lý Hermite)}$
$\Rightarrow S_n=\left\lfloor x\right\rfloor-\left\lfloor \dfrac{n}{2^{n+1}}\right\rfloor$
Với $n$ đủ lớn thì $2^{n+1}>n$ hay $\left\lfloor \dfrac{n}{2^{n+1}}\right\rfloor=0$
Từ đó suy ra điều phải chứng minh

#26
hxthanh

hxthanh

    Tín đồ $\sum$

  • Hiệp sỹ
  • 3921 Bài viết
$$\begin{array}{|c|}
\hline
\fbox{BỔ ĐỀ II}\\
\sum\limits_{k=a}^b \Big(\left\lfloor f(k)\right\rfloor+\left\lfloor f(-k)\right\rfloor\Big) = a-b-1+\underset{[a,\;b]}{G}\Big(f(k)\Big)\\
\begin{array}{l}
\text{Trong đó }\; \underset{[a,\;b]}{G}\Big(f(k)\Big) \; \text{là số điểm nguyên của đồ thị }\; (G)\; \text{ của hàm } \;f(k) \text{ trên đoạn }\; [a,\;b]\\
\end{array}\\
\hline
\end{array}$$
Phát biểu của bổ đề khá là đơn giản nhưng lại rất có ích trong việc tính tổng phần nguyên.
Vì: $\left\lfloor x\right\rfloor+\left\lfloor -x\right\rfloor= \begin{cases}&0& \quad\text{Với }x \in\mathbb{Z} \\ &-1& \quad\text{Với }x \notin\mathbb{Z} \end{cases}$

Nếu coi như đồ thị của $f(k)$ không có điểm nguyên nào trên $[a,\;b]$ thì

$VT=\sum\limits_{k=a}^b (-1) = a-b-1$

Thực tế mỗi điểm nguyên của đồ thị $f(k)$ trên đoạn $[a,\;b]$ sẽ làm cho tổng đó tăng lên $1$
Do đó ta có điều phải chứng minh.
________________________________

Hãy áp dụng bổ đề trên để tính tổng:
$S=\sum\limits_{k=1}^n \left\lfloor-\dfrac{k}{m}\right\rfloor$

#27
4869msnssk

4869msnssk

    Bá tước

  • Thành viên
  • 549 Bài viết

Bài này  +) x là số vô tỉ thì $x^{3}-\left [ x \right ]$ là số vô tỉ (L)

+) x là số hữu tỉ thì $x^{3}-\left [ x \right ]$ là số hữu tỉ (L)

$\Rightarrow x\epsilon \mathbb{N}$$\Rightarrow x^{3}-x=3\Leftrightarrow x(x-1)(x+1)=3(L)$ 

Phương trình vô nghiệm

cái này thì chưa khẳng định ngay đc, bạn hãy cm cụ thể ra


 B.F.H.Stone


#28
hxthanh

hxthanh

    Tín đồ $\sum$

  • Hiệp sỹ
  • 3921 Bài viết

Bt2.18 là một sự nhầm lẫn đó mọi người (tổng như vậy không tính được!)

Đề đúng phải là: $\displaystyle S=\sum_{k=1}^n \frac{k}{m} $

Xin lỗi mọi người vì sự cố trên. :)



#29
perfectstrong

perfectstrong

    $LOVE(x)|_{x =\alpha}^\Omega=+\infty$

  • Quản lý Toán Ứng dụng
  • 4996 Bài viết

Bt2.18 là một sự nhầm lẫn đó mọi người (tổng như vậy không tính được!)

Đề đúng phải là: $\displaystyle S=\sum_{k=1}^n \frac{k}{m} $

Xin lỗi mọi người vì sự cố trên. :)

Thế thì giống cái này hả thầy?

 

Để tính toán các tổng phần nguyên hữu tỉ, ta có một BỔ ĐỀ khá quan trọng sau:
$$\begin{array}{| |}
\hline
\boxed{\text{ BỔ ĐỀ I }}\\
\text{Với mọi số nguyên dương } m \text{ và } n \text{ ta có: }\\
\boxed{\sum\limits_{k=0}^n \left\lfloor\dfrac{k}{m}\right\rfloor=\left(n+1-\dfrac{m}{2}\right)\left\lfloor\dfrac{n}{m}\right\rfloor-\dfrac{m}{2}\left\lfloor\dfrac{n}{m}\right\rfloor^2}\\
\hline
\end{array}$$
Chứng minh:
Với $m=1$: Bổ đề trở thành công thức quen thuộc $\sum\limits_{k=0}^n k =\dfrac{n(n+1)}{2}$.
Xét với $m \ge 2$

Ta sẽ nhóm các số hạng theo số thứ tự đồng dư modul $m$ bằng cách đặt
$k=pm+r,\quad (0 \le r \le m-1)$
Khi đó $0 \le p \le \left\lfloor\dfrac{n}{m}\right\rfloor-1$, phần "dôi ra" sẽ chạy từ $k=m\left\lfloor\dfrac{n}{m}\right\rfloor $ đến $k=n$


Ta có:
$\sum\limits_{k=0}^n \left\lfloor\dfrac{k}{m}\right\rfloor=\sum\limits_{p=0}^{\left\lfloor\frac{n}{m}\right\rfloor-1}\sum\limits_{r=0}^{m-1} \left\lfloor\dfrac{pm+r}{m}\right\rfloor+\sum\limits_{k=m\left\lfloor\frac{n}{m}\right\rfloor}^n \left\lfloor\dfrac{k}{m}\right\rfloor=\sum\limits_{p=0}^{\left\lfloor\frac{n}{m}\right\rfloor-1} mp + \left(n+1-m\left\lfloor\dfrac{n}{m}\right\rfloor\right)\left\lfloor\dfrac{n}{m}\right\rfloor$
$\qquad\qquad = m\left(\dfrac{1}{2}\left(\left\lfloor\dfrac{n}{m}\right\rfloor-1\right)\left\lfloor\dfrac{n}{m}\right\rfloor\right)+(n+1)\left\lfloor\dfrac{n}{m}\right\rfloor-m\left\lfloor\dfrac{n}{m}\right\rfloor^2$
$\qquad\qquad=\left(n+1-\dfrac{m}{2}\right)\left\lfloor\dfrac{n}{m}\right\rfloor-\dfrac{m}{2}\left\lfloor\dfrac{n}{m}\right\rfloor^2$
Vậy ta có điều phải chứng minh.
$\square$
Sau đây ta sẽ áp dụng BỔ ĐỀ này để giải quyết Bài toán $\boxed{Bt2.11}$


Luôn yêu để sống, luôn sống để học toán, luôn học toán để yêu!!! :D
$$\text{LOVE}\left( x \right)|_{x = \alpha}^\Omega = + \infty $$
I'm still there everywhere.

#30
ducthang0701

ducthang0701

    Hạ sĩ

  • Thành viên
  • 79 Bài viết

không biết làm thế nào để tải về nhỉ



#31
frozen2501

frozen2501

    Binh nhất

  • Thành viên mới
  • 49 Bài viết

tìm nghiệm nguyên của phương trình: (x^2 + y^2 +1)^2 -5x^2 -4y^2 -5=0


Every thing will be alright


#32
hoakute

hoakute

    Trung sĩ

  • Thành viên
  • 149 Bài viết

$(x^{2}+y^{2}+1)^{2}-5(x^{2}+y^{2}+1)=-y^{2} \Leftrightarrow (x^{2}+y^{2}+1)(x^{2}+y^{2}-4)=-y^{2} \Rightarrow (x^{2}+y^{2}+1)(x^{2}+y^{2}-4)\leqslant 0\Leftrightarrow -1\leqslant x^{2}+y^{2}\leqslant 4\Rightarrow x^{2}+y^{2}\epsilon \left \{ 0;1;2;3;4 \right \}$

Mà x2 và y2 là các scp.

Suy ra x2=0;1;4 tương ứng y2=4;0;1 hoặc 4;1;0

Tìm đc x=+-2; y=0



#33
lenadal

lenadal

    Trung sĩ

  • Thành viên
  • 161 Bài viết

tìm nghiệm nguyên của phương trình: (x^2 + y^2 +1)^2 -5x^2 -4y^2 -5=0

ta có phương trình đó tương đương với 

$x^{4}+y^{4}+1+2x^{{2}}y^{2}+2x^{2}+2y^{2}-5x^{2}-4y^{2}-5=0$

$\Leftrightarrow (x^{2}+y^{2})^{2}-2(x^{2}+y^{2})+1-x^{2}=5$

$\Leftrightarrow (x^{2}+y^{2}-1-x)(x^{2}+y^{2}-1+x)=5$

tự làm tiếp!!!!


Lê Đình Văn LHP    :D  :D  :D 

http://diendantoanho...150899-lenadal/


#34
DangHongPhuc

DangHongPhuc

    Thiếu úy

  • Thành viên
  • 657 Bài viết

Theo định lý Legendre thì số mũ cao nhất của $19$ có trong $(1994)!$ là:

Thầy ơi cho em hỏi định lý Legendre phát biểu như thế nào ạ?


"Con người không sợ Thần

mà bản thân nỗi sợ chính là Thần"


#35
VyHuynh

VyHuynh

    Lính mới

  • Thành viên mới
  • 7 Bài viết

Bài 1: Sử dụng định lý Lagrande về số mũ của cao nhất của một số nguyên tố chứa trong n!

Ta có $1995=3.5.7.19$

Theo định lý Legendre thì số mũ cao nhất của $19$ có trong $(1994)!$ là:

$\left\lfloor\dfrac{1994}{19}\right\rfloor+\left\lfloor\dfrac{1994}{(19)^2}\right\rfloor+...+\left\lfloor\dfrac{1994}{(19)^k}\right\rfloor+...=109$

Như vậy $(1994)!\;\vdots\; (1995)^{109}$ và $(1994)!\;\not{\vdots} \;(1995)^{n\ge110}$

Suy ra để $((1994)!)^{1995}\;\vdots\; (1995)^k$ thì $k\le 109*1995=217\;455$



#36
Thanh Long 2001

Thanh Long 2001

    Binh nhất

  • Thành viên mới
  • 32 Bài viết

GIúp mình. :wacko:  :wacko:



#37
nhatkinan

nhatkinan

    Hạ sĩ

  • Thành viên
  • 56 Bài viết

mk có tài liệu phần nguyên nè

 

File gửi kèm



#38
nhatkinan

nhatkinan

    Hạ sĩ

  • Thành viên
  • 56 Bài viết

cảm ơn mọi người đóng góp nha



#39
vuthanhninh3105

vuthanhninh3105

    Lính mới

  • Thành viên mới
  • 6 Bài viết

Hai bài phần nguyên trong kỳ thi của ĐHKHTN các năm trước
$\begin{array}{| |}
\hline
&\boxed{\text{Bài 1}}&\\
&\text{Chứng minh rằng với mọi } n \text{ tự nhiên thì:}&\\
&\left\lfloor\sqrt[3]{72n+1}\right\rfloor=\left\lfloor\sqrt[3]{9n}\right\rfloor+\left\lfloor\sqrt[3]{9n+1}\right\rfloor=\left\lfloor\sqrt[3]{72n+7}\right\rfloor&\\
&&&\\
\hline
\end{array}$

$\begin{array}{| |}
\hline
&\boxed{\text{Bài 2}}&\\
&\text{Chứng minh rằng với mọi số nguyên dương } n \text{ thì:}&\\
&A=n+\left\lfloor\sqrt[3]{n-\dfrac{1}{27}}+\dfrac{1}{3}\right\rfloor^2&\\
&\text{không thể là một lập phương đúng!}&
&&&\\
\hline
\end{array}$

Cho mình hỏi 2 bài trên thi năm nào đấy?



#40
TenLaGi

TenLaGi

    Trung sĩ

  • Thành viên
  • 120 Bài viết

Đây là bài phần nguyên thi thử ĐHKHTN đợt 3 năm nay. MN tham khảo nha: CMR $\begin{bmatrix} & \sqrt{n}+\frac{1}{2} \end{bmatrix}$=$\begin{bmatrix} & \sqrt{n+\sqrt{n}} \end{bmatrix}$


            ~~~Chữ tâm kia mới bằng ba chữ tài~~~

                         





2 người đang xem chủ đề

0 thành viên, 2 khách, 0 thành viên ẩn danh